googled combined phys222 sets

Pataasin ang iyong marka sa homework at exams ngayon gamit ang Quizwiz!

Which of the following are true of static charges on everyday objects? (a) Like charges repel. (b) Like charges attract. (c) Opposite charges repel. (d) Opposite charges attract. (e) A positively charged object has lost electrons. (f) A positively charged object has gained protons. (g) A negatively charged object has lost protons. (h) A negatively charged object has gained electrons.

(a), (d), (e), (h) only

Which of the following are true of static charges on everyday objects? (a) Like charges repel. (b) Like charges attract. (c) Opposite charges repel. (d) Opposite charges attract. (e) A positively charged object has lost electrons. (f) A positively charged object has gained protons. (g) A negatively charged object has lost protons. (h) A negatively charged object has gained electrons. Answers: (a), (d), (e), (h) only (a), (d), (e), (f), (g), (h) only (a) and (d) only (a), (d), (f), (g) only (b), (c), (f), (g) only (c) and (h) only

(a), (d), (e), (h) only

Which of the diagrams represents resistors connected in series? IMAGE REVIEW PROBLEMS Answers: (a) and (b) only all diagrams (b) only (a) only (c) only

(b) only

Which of the following statements are true about the electric field? (a) The strength of the electric field created by an object A at a point P increases by a factor of 4 is the charge on object A is doubled. (b) The strength of the electric field created by an object A at a point P decreases by a factor of 4 is the distance between A and P is doubled (c) The strength of the electric field created by an object A at a point P is unchanged if both the charge on object A and the distance between A and P are doubled.. (d) The strength of the electric field inside a conductor is zero. (e) For irregularly shaped conductors, the electric field strength is greatest around points of highest curvature. (f) The SI unit for the electric field is N/C.

(b), (d), (e), (f), only

In the early universe and in stars, deuterium nuclei are produced from the combination of one proton and one neutron, with the release of a gamma ray. What is the charge on a deuterium nucleus in units of qe = 1.6*10-19 C? -1 -2 2 1 0

+1

Each of three objects has a net charge. Objects A and B attract one another. Objects B and C also attract one another, but objects A and C repel one another. Which one of the following table entries is a possible combination of the signs of the charges on these three objects? A B C Answers: - - + + - - - + - + + - - + +

- + -

In the hydrogen atom, the total energy of electron with a very large principal quantum number n is approximately

0

Two narrow slits spaced 100 micrometers apart are exposed to light of 600 nm. At what angle does the first minimum (dark space) occur in the interference pattern? Answers: 0.003 radians 0.344 radians 0.006 degrees 0.006 radians 0.003 degrees

0.003 radians

Electrons with energy of 25 eV have a wavelength of ~0.25 nm. If we send these electrons through the same two slits (d = 0.42 mm) we use to produce a visible light interference pattern what is the spacing (in micrometer) between maxima on a screen 3.5 m away?

0.25x10^-9 x 3.5 / 0.42x10^-3= 2.08

A gamma ray photon has an energy of 1.95 GeV. (1 GeV = 109 eV.) What is the wavelength of the gamma ray in fm? (1 nm = 10-15 m)?

0.636

In 7,780 seconds, the number of radioactive nuclei decreases to 1/16 of the number present initially. What is the half-life (in s) of the material? Enter an integer.

1,945

Choose the correct statement. 1. Nearly the entire mass of the atom is concentrated in the nucleus and the mass of the electrons is almost negligible. 2. Nearly the entire charge of the atom is concentrated in the nucleus and the charge of the electrons is almost negligible. 3. STM images reveal the distribution of both the negative and the positive charge in an atom. 4. Mass is uniformly distributed throughout the atom and hence most alpha-particles are deflected in the backward direction in alpha - particle scattering experiments.

1. Nearly the entire mass of the atom is concentrated in the nucleus and the mass of the electrons is almost negligible.

Which of the following is not true? 1. The nuclear force and the electric force between two protons close to each other have approximately the same magnitude. 2. The nuclear force has a short range, of the order of nuclear dimensions. 3. A nucleon in a large nucleus interacts via the nuclear force only with nearby nucleons, not with ones far away in the nucleus. 4. The nuclear force does not depend on the electric charge.

1. The nuclear force and the electric force between two protons close to each other have approximately the same magnitude.

How does the binding energy per nucleon of a fusion product compare to that of the pieces that combined to form it? 1. The product has a greater binding energy per nucleon than the pieces. 2. The product has less binding energy per nucleon than the pieces. 3. The product has the same binding energy per nucleon than the pieces. 4. It depends on which exact reaction, i.e. on which pieces.

1. The product has a greater binding energy per nucleon than the pieces.

On a balance scale, you put 2 neutrons and 1 proton on one side and you put a tritium nucleus (3H) on the other. Which side weighs more? 1. The two neutrons and 1 proton. 2. The tritium nucleus. 3. Both sides weigh the same. 4. It depends on the specific tritium isotope.

1. The two neutrons and 1 proton.

If the wavelength of the light were changed, and yellow light were passed through the slit, 1. the central bright region would become wider. 2. the central bright region would become narrower. 3. the width of the central bright region would not change, but it would appear dimmer. 4. The width of the central bright region would not change, but it would appear even brighter.

1. the central bright region would become wider.

When a car is 500 m ahead of you, you see its tail lights as one long, red light. When the car is 100 m ahead of you, you see that the tail lights are actually several red lights placed close to each other. This is because 1. the pupil of your eye has a finite size. 2. the Doppler effect for light shifts the frequency of the tail lights. 3. light disperses in the lens of your eye 4. light is made up of photons. 5. light is polarized.

1. the pupil of your eye has a finite size.

The principal quantum number 'n' of an electron in an atom is primarily 1. used to identify the main energy level or shell. 2. used to determine the orientation of the orbital. 3. used to determine the shape of the orbital.

1. used to identify the main energy level or shell.

Electrons with energy of 25 eV have a wavelength of ~0.25 nm. If we send these electrons through the same two slits (d = 0.41 mm) we use to produce a visible light interference pattern what is the spacing (in micrometer) between maxima on a screen 2.3 m away?

1.4

What is the deBroglie wavelength of a beam of "cars", having a mass of 1500 kg and a speed of 90 km/h?

1.8*10-38 m

Electrons are ejected from a metal surface with energies ranging up to 0.5 eV when light with a wavelength of 529 nm is used. What is the work function of the metal in units of eV?

1.84

The radius of an atomic nucleus is of the order of

10 -15 m.

A thin film of oil (n = 1.51) is located on a smooth, wet pavement. When viewed perpendicular to the pavement, it appears to be predominantly red (641 nm). What is the minimum thickness of the oil film in nm?

106.1

The radius of an atomic nucleus is of the order of Answers: a. 10^-18 m. b. 10^-8 m. c. The nucleus is a point and has no radius. d. 10^-15 m. e. 10^-12 m. f. 10^-10 m.

10^-15 m.

The half-life of 14C is 5730 years. Suppose an ancient bone is found in which the ratio of 14C /12C is 1/4 that found in the atmosphere and in living plants and animals. Approximately how old is the bone?

12000 years

The half-life of 14C is 5730 years. Suppose an ancient bone is found in which the ratio of 14C /12C is 1/4 that found in the atmosphere and in living plants and animals. Approximately how old is the bone? Answers: 1500 years 3000 years 9000 years 12000 years 6000 years

12000 years

What is the approximate mass number (number of nucleons) of a nucleus whose radius is measured to be 6*10-15 m?

125

If the orbital angular momentum is l = 7, how many different projections along the z-axis are possible?

15

Which of the following nuclei likely has the largest binding energy per nucleon?

168O

A hydrogen atom electron is excited to an energy of −13.6/9 eV. How many different quantum states are there with this energy?

18

Assume that in an atom the subshell n = 5, l = 4 is completely filled. How many electrons does this subshell hold?

18

You have 16 kg of a radioactive sample with a certain half-life of 15 years. How much is left after 60 years?

1kg

Which of the following condensed notations gives the lowest energy electronic configuration for doubly ionized Magnesium (Mg)? p1.gif

1s2 2s2 2p6

Which of the following condensed notations gives the lowest energy electronic configuration for doubly ionized Magnesium (Mg)? Answers: a. 1s2 2s2 2p6 3s2 3p6 b. 1s2 2s2 2p4 3s2 c. 1s2 2s2 2p6 3s2 d. 1s2 2s2 2p6

1s2 2s2 2p6

What is the approximate mass number (number of nucleons) of a nucleus whose radius is measured to be 6*10-15 m? 1. 6 2. 125 3. 256 4. 80 5. 64

2. 125

Why is this common picture of a neutral atom wrong? (IMAGE: Nov 12) A. The relative scale is wrong. B. The pictured orbits violate the uncertainty principle. C. No matter how you look at it, this atom is not neutral. 1. A only 2. A and B only 3. B and C only 4. A,B,andC

2. A and B only

The figure shows the interference pattern obtained in a double-slit experiment using light of wavelength 500 nm. distance from both slits? Which fringe is the same IMAGE OCT 29 1. A 2. B 3. C 4. D 5. E

2. B

Radionuclides used in PET scanning are typically isotopes with short half-lives, for example carbon-11 (~20 min). Carbon-11 decays, and a Boron-11 nucleus is left behind. What type of decay is this? 1. Alpha decay 2. Beta plus decay 3. Beta minus decay 4. Gamma decay 5. Proton emission 6. Neutron emission

2. Beta plus decay

What is the first element in the periodic table to have a filled 4s subshell? 1. Al 2. Ca 3. Mg 4. Ar

2. Ca

Which of the statements below is correct for a photon. 1. Given the exact momentum of a photon, you can calculate its exact position. 2. Given the exact momentum of a photon, you know nothing about its position. 3. Given that a photon is emitted by a source and absorbed by a target after a time interval 1 ns, you know its travel time and therefore its exact energy.

2. Given the exact momentum of a photon, you know nothing about its position.

The long-range electrostatic repulsion between protons limits the size of stable nuclei. Why are there no large nuclei consisting only of neutrons, which do not repel each other? 1. The nuclear force acting on protons is stronger than that acting on neutrons, so neutrons would not be bound. 2. The Pauli exclusion principle would require the neutrons to occupy very high energy states, yielding the nucleus unstable. 3. Nuclei are in the center of atoms, and the atomic electrons would not be bound if there were no protons in the nucleus.

2. The Pauli exclusion principle would require the neutrons to occupy very high energy states, yielding the nucleus unstable.

Extra Credit: How does the total mass of the fission fragments compare to the mass of the original nucleus in a fission reaction? 1. The fission fragments have more mass than the original nucleus. 2. The fission fragments have less mass than the original nucleus. 3. The fission fragments have the same mass as the original nucleus. 4. Not enough information is given to decide.

2. The fission fragments have less mass than the original nucleus.

Extra Credit: Consider a double slit interference experiment with electrons. What happens to the interference pattern if we increase the momentum of the electrons? 1. Nothing 2. The fringes get closer together. 3. The fringes get further apart.

2. The fringes get closer together.

An electron with an energy of 1000 eV has a deBroglie wavelength of 0.04 nm. If the speed of the electron increases by a factor of 2, the wavelength will 1. increase by a factor of 2. 2. decrease by a factor of 1/2. 3. increase by a factor of 4. 4. decrease by a factor of 1/4.

2. decrease by a factor of 1/2.

A radioactive atom X emits a β− particle. The resulting atom 1. must be very reactive chemically. 2. has an atomic number that is one more than that of X. 3. has a mass number that is one less than that of X. 4. must be radioactive. 5. is the same chemical element as X.

2. has an atomic number that is one more than that of X.

The 'magnetic' quantum number 'm' of an electron in an atom is primarily 1. used to identify the main energy level or shell. 2. used to determine the orientation of the orbital. 3. used to determine the shape of the orbital.

2. used to determine the orientation of the orbital.

Light of wavelength λ1 illuminates a double slit, and interference fringes are observed on a screen behind the slits. When the wavelength is changed to λ2, the fringes move closer together. What is the relationship between λ1 and λ2. 1. λ1 is smaller than λ2. 2. λ1 is larger than λ2. 3. A relationship cannot be determined from the given information.

2. λ1 is larger than λ2.

A laser emits photons continuously at a rate of 2*1016/s. If the wavelength of the photons is 637 nm, what is the laser power in mW?

2.0*10^16((6.626*10^-34)*(3*10^8)/(637*10^-9))

The energy of a photon that has a wavelength of 9.0 m is __________ J. Answers: 6.0*10-23 2.7*109 4.5*10-25 2.2*10-26 4.5*1025

2.2*10-26

The energy of a photon that has a wavelength of 9.0 m is _______ J.

2.2x10-26 ((6.63 * 10 ^-34) (3 *10 ^ 8) )/ 9 = 2.2 x 10 ^-26

A 1 ns laser pulse from a neodymium-YAG laser, at 532 nm, contains 5 J. What is the energy of each photon in the pulse in units of eV? (1 eV = 1.6*10-19 J) Answers: 660 keV 0.43 eV 3.74*10-19 eV 2.33 eV 5.64*1014 eV

2.33 eV

A 1 ns laser pulse from a neodymium-YAG laser, at 532 nm, contains 5 J. What is the energy of each photon in the pulse in units of eV? (I eV = 1.6 x 10-19 J)

2.33 eV

What is the de Broglie wavelength of a beam of "cars", having a mass of 1500 kg and a speed of 65 km/h? Answers: 6.8*10-39 m 6.8*10-10 m 2.45*10-38 m 27083 m 6.8*10-42 m

2.45*10-38 m

For a grating, interference maxima are observed at angles θ, for which d sinθ = mλ. If a grating has 4000 grooves per cm, what is d?

2.5 micrometer

210Po (Polonium) is an unstable nucleus which alpha decays. The resulting nucleus is

206Pb

228(90)Th emits a gamma ray. What is the resulting nucleus? Answers: 228(91)Pa 230(92)U 229(90)Th 228(90)Th 232(92)U

228(90)Th

22890Th emits a gamma ray. What is the resulting nucleus?

228/90Th

The isotope 23492U undergoes five successive alpha decays. Identify the correct decay chain.

23090Th --> 22688Ra --> 22286Rn --> 21884Po --> 21482Pb

The isotope 23492U undergoes five successive alpha decays. Identify the correct decay chain. Answers: a. 23090Th --> 22688Ra --> 22286Rn --> 21884Po --> 21482Pb b. 23290Th --> 22888Ra --> 22886Rn --> 22084Po --> 21682Pb c. 23390Th --> 22988Ra --> 22586Rn --> 21884Po --> 21782Pb d. 22890Th --> 22888Ra --> 22886Rn --> 21884Po --> 21282Pb

23090Th --> 22688Ra --> 22286Rn --> 21884Po --> 21482Pb

Suppose a Uranium 238 nucleus "swallows" a neutron and then decays not by fission but by beta decay, emitting an electron and an anti-neutrino. Which nucleus remains after this decay?

239Np

Suppose a Uranium 238 nucleus "swallows" a neutron and then decays not by fission but by beta decay, emitting an electron and an anti-neutrino. Which nucleus remains after this decay? Answers: 238Np 239U 239Np 239Pu 238Pa

239Np

At what rate must Uranium 235 undergo fission by neutron bombardment to generate energy at a rate of 783 W (1W = 1J/s)? Assume each fission reaction releases 200 MeV of energy.

24,500,000,000,000

What is the ratio of the longest wavelength of the Balmer series to the longest wavelength of the Lyman series? Answers: 5/27 27/5 9/4 3/2 4/9 2/3

27/5

Blue light of wavelength λ passes through a single slit of width a and forms a diffraction pattern on a screen. If the blue light is replaced by red light of wavelength 2λ, the original diffraction pattern is reproduced if the slit width is changed to what?

2a

Blue light of wavelength λ passes through a single slit of width a and forms a diffraction pattern on a screen. If the blue light is replaced by red light of wavelength 2λ, the original diffraction pattern is reproduced if the slit width is changed to what? Answers: No change is necessary. a/2. a/4. 4a 2a

2a

A hypothetical atom has 3 energy states. Assuming all transitions are possible, how many spectral lines can this atom produce?

3

How many of following statements about the photoelectric effect are true? (a) The greater the frequency of the incident light is, the greater is the stopping potential. (b) The greater the intensity of the incident light is, the greater is the cutoff frequency. (c) The greater the work function of the target material is, the greater is the stopping potential. (d) The greater the work function of the target material is, the greater is the cutoff frequency. (e) The greater the frequency of the incident light is, the greater is the maximum kinetic energy of the ejected electrons. (f) The greater the energy of the photons is, the smaller is the stopping potential.

3

Which of the following best describes the image formed by a plane mirror? (1) virtual, inverted and enlarged (2) real, inverted and reduced (3) virtual, upright and the same size as object (4) real, upright and the same size as object Answers: 1 2 3 4

3

An excited hydrogen electron is in the principal shell n = 3. Which of the following is a possible set of quantum numbers (n, l, m, ms) for the electron's state? Answers: 3, 1, 2, 1/2 3, 2, -3, 1/2 3, 1, 2, 1/2 3, 2, -2, -1/2 none of the above

3, 2, -2, -1/2

An excited hydrogen electron is in the principal shell n = 3. Which of the following is a possible set of quantum numbers (n, l, m, ms) for the electron's 4state?

3, 2, -2, -½

A hypothetical atom has 3 energy states. Assuming all transitions are possible, how many spectral lines can this atom produce? 1. 1 2. 2 3. 3 4. 4

3. 3

If the orbital angular momentum quantum number l in a hydrogen atom were allowed to take values from 0 to n (instead of n - 1, then number of electrons in the n = 1 shell would be 1. 2. 2. 6. 3. 8. 4. 10. 5. 18.

3. 8.

Which of the following is true? 1. Physicists aren't really sure what predictions Quantum Mechanics makes about experiments. 2. Quantum Mechanics is not an exact theory, because it only gives approximate predictions about the results of experiments. 3. After a measurement we can know the exact energy of a system. But there will be a spread in outcomes if we subsequently measure the position. 4. The Uncertainty Principle would not apply if we knew the exact initial wave function, Ψ(x,t=0), for the particle.

3. After a measurement we can know the exact energy of a system. But there will be a spread in outcomes if we subsequently measure the position.

What is the definition of the of nuclear binding energy? 1. It is the amount of energy needed to remove the most loosely bound electron from an atom. 2. It is the amount of energy needed to remove the most loosely bound proton from a nucleus. 3. It is the amount of energy needed to completely separate a nucleus into its component neutrons and protons. 4. It is the amount of energy needed to remove all electrons from an atom. 5. It is the amount of energy needed to remove all electrons from an atom and then take the nucleus apart.

3. It is the amount of energy needed to completely separate a nucleus into its component neutrons and protons.

Assume that in the experiment shown below electrons are ejected with some kinetic energy when violet light hits a metal surface. Which of the following changes to the experiment would decrease the maximum kinetic energy of the ejected electrons. 1. Using a more intense light beam 2. Using a less intense light beam 3. Using blue light 4. Using UV light 5. None of the above

3. Using blue light

An electron with an energy of 1000 eV has a de Broglie wavelength of 0.04 nm. If the energy of the electron decreased by a factor of 2, the wavelength will 1. increase by a factor of 2. 2. decrease by a factor of 1/2. 3. increase by a factor of √2. 4. decrease by a factor of 1/√2.

3. increase by a factor of √2.

Which of the transitions below can produce a spectral line in the visible region of the emission spectrum of hydrogen? (IMAGE: Nov 12) 1. n = 2 n = 5 2. n = 3 n = 1 3. n = 4 n = 2 4. n = 5 n = 3

3. n = 4 n = 2

The energy required to remove an electron from a hydrogen atom, if the electron is in the 1st excited state, is Answers: 0.85 eV. The hydrogen has only one electron, and it is impossible to remove it completely. 13.6 eV. 3.4 eV. 1.5 eV.

3.4 eV.

A stream of electrons is of energy E is incident on a potential barrier of height U and thickness d. Even though U >> E, 5% of the electrons tunnel through the barrier. If the thickness of the barrier decrease to 0.35 d, what percentage of the electrons will tunnel through? Enter an integer.

35

An electron in a hydrogen atom makes a transition from the n = 2 to the n = 28 energy state. Determine the wavelength of the emitted photon (in nm).

367

An electron in a hydrogen atom makes a transition from the n = 8 to the n = 2 energy state. Determine the wavelength of the emitted photon (in nm).

389

A group of quantum state of the hydrogen atom has n = 4. How many values of l are possible for the states in this group?

4

A group of quantum state of the hydrogen atom has n = 4. How many values of l are possible for the states in this group? Answers: 4 5 3 2 1 6 0

4

A group of quantum state of the hydrogen atom has n = 4. How many values of l are possible for the states in this group? a) 3 b) 1 c) 6 d) 5 e) 4 f) 0

4

A partial energy level spectrum of hypothetical element X is given below. Of the transitions listed below, which one produces the shortest wavelength photon? IMAGE PRACTICE 1 Answers: 6 --> 5 4 --> 1 4 --> 3 6 --> 2 2 --> 1

4 --> 1

A partial energy level spectrum of hypothetical element X is given below. Of the transitions listed below, which one produces the shortest wavelength photon?

4 → 1

What is the deBroglie wavelength of a beam of "cars", having a mass of 1500 kg and a speed of 90 km/h? 1. 13500 m 2. 37500 m 3. 2.7*10-5 m 4. 1.8*10-38 m 5. 6.8*10-10 m

4. 1.8*10-38 m

A hydrogen atom electron is excited to an energy of −13.6/9 eV. How many different quantum states are there with this energy? Hint: Find n, find all possible l, for each l, find all possible m and ms. Remember: n = 1, 2, 3, ... l = 0, 1, 2, ... n-1 m = 0, ±1, ±2, ... ±l ms = ± 1⁄2 1. 1 2. 4 3. 8 4. 18 5. More than 18

4. 18

You have 16 kg of a radioactive sample with a certain half-life of 15 years. How much is left after 60 years? 1. 8kg 2. 4kg 3. 2kg 4. 1kg 5. nothing

4. 1kg

For a grating, interference maxima are observed at angles θ, for which d sinθ = mλ. If a grating has 4000 grooves per cm, what is d? 1. 4000 cm 2. 2 micrometer 3. 0.25 mm 4. 2.5 micrometer 5. 5 nanometer

4. 2.5 micrometer

Extra Credit: 210Po (Polonium) is an unstable nucleus which alpha decays. The resulting nucleus is 1. 210Pb 2. 206Po 3. 214Rn 4. 206Pb 5. 206Rn

4. 206Pb

Photons with λ = 250 nm (hf = 5.0 eV) hit a metal surface and emit photoelectrons with a maximum kinetic energy Eelectron = 1.0 eV. What is the work function of the metal? 1. 1eV 2. 2eV 3. 3eV 4. 4eV 5. 5eV

4. 4eV

What wavelength monochromatic source in the visible region (390 to 710 nm) can be used to constructively reflect off a coating on glass (n = 1.46), if the coating is 191 nm thick? 1. 409 nm 2. 430 nm 3. 450 nm 4. 558 nm 5. 627 nm

4. 558 nm

Extra Credit: A telescope with a 6 m diameter lens is in orbit around Earth. What is its maximum resolution for 400 nm light, i.e. what is the smallest angle θ(rad) between objects it just can resolve? 1. 4*10-9 2. 6*10-7 3. 1.22*10-7 4. 8.13*10-8

4. 8.13*10-8

The blue tint of a coated camera lens is largely caused by what effect? 1. Diffraction 2. Refraction 3. Polarization 4. Interference 5. Absorption

4. Interference

Extra Credit: To sustain laser action it is usually necessary to place the lasing material between the two mirrors of an optical cavity. Why? 1. This is required by the uncertainty principle. 2. We have to confine the photons, because confinement leads to energy quantization. 3. The Pauli exclusion principle requires that the photons have different quantum numbers. The cavity guaranties this. 4. The cavity recycles most of the photons and therefore increases the probability of duplication.

4. The cavity recycles most of the photons and therefore increases the probability of duplication.

Consider both the nuclear and the electrostatic interaction. A proton in a very large nucleus 1. attracts all other protons. 2. repels all other protons. 3. repels all neutrons. 4. attracts some protons and repels others. 5. attracts some neutrons and repels others.

4. attracts some protons and repels others.

We observe an interference patterns using light. This suggests that light 1. has a phase of 180o. 2. is a transvers wave. 3. is a stream of photons. 4. behaves like a wave. 5. has electric properties.

4. behaves like a wave.

An energy eigenfunction ψ(x) for an electron in the potential energy well pictured below is shown. Around which point is the electron most likely to be found? (IMAGE: NOV 5) 1. x= A 2. x= B 3. x= C 4. x= D 5. There is no most likely place.

4. x= D

A laser emits photons continuously at a rate of 1.1*1016/s. If the wavelength of the photons is 520 nm, what is the laser power in mW?

4.2

A hydrogenic He ion is excited from its ground state to the state with n = 2. How much energy (in eV) must be absorbed by the ion?

40.8

A hydrogenic He ion is excited from its ground state to the state with n = 2. How much energy (in eV) must be absorbed by the ion? Answers: 10.2 27.2 54.4 20.4 13.6 40.8

40.8

What is the wavelength (in nm) of a photon with energy E = 3 eV

413

A Young'sdouble-slit interference experiment is performed with monochromatic light. The separation between the slits is 0.33 mm. The interference pattern on the screen 4.3 m away shows the first maximum 5.7 mm from the center of the pattern. What is the wavelength of the light in nm?

437

The angular separation of the two components of a double star is 1.5 microradians, and the light from the double star has a wavelength of 561 nm. What is the diameter (in cm) of the smallest telescope mirror that will resolve the double star?

46

Light from an argon laser strikes a diffraction grating that has 4,601 lines per cm. The central and first-order principal maxima are separated by 0.4 m on a wall 1.77 m from the grating. What is the wavelength of the laser light in nm?

479

Photons with λ = 250 nm (hf = 5.0 eV) hit a metal surface and emit photoelectrons with a maximum kinetic energy Eelectron = 1.0 eV. What is the work function of the metal?

4eV

Two parallel light rays, initially in phase and having a 500 nm wavelength, reach a detector after one of the rays travels through a 10 cm long block of glass with an index of refraction of 1.5, while the other ray stays in air. The optical path difference between the two rays at the detector is

5 cm

In the hydrogen atom, the total energy of electron with a very large principal quantum number n is approximately 1. -13.6 eV. 2. -3.4 eV. 3. -1.51 eV. 4. -0.85 eV. 5. 0.

5. 0.

If the orbital angular momentum is l = 7, how many different projections along the z-axis are possible? 1. 7 2. 9 3. 13 4. 14 5. 15

5. 15

Which of the following nuclei likely has the largest binding energy per nucleon? 1. 15(7)N 2. 14(7)N 3. 12(6)C 4. 13(6)C 5. 16(8)O

5. 16(8)O

If the orbital angular momentum is l = 7, what is the magnitude of the orbital angular momentum, and what is the largest value its projection can have along any axis in units of ħ? 1.56, 6 2.7, 6 3.7, 7 4. 6.48, 6 5. 7.48, 7

5. 7.48, 7

An ideal polarizer will transmit 50% of unpolarized light incident on it. How much light is transmitted by two ideal polarizers in series that are oriented with their transmission axes parallel to each other? Answers: between 50% and 100% between 0% and 50% 0% 100% 50%

50%

Unpolarized light is incident on a linear polarizer. What fraction of the light is transmitted? a) 100% b) 0% c) It depends on the orientation of the polarizer's transmission axis. d) 50%

50%

Light incident on a 0.15 mm wide slit forms a diffraction pattern on a screen 6 m away. If the first minimum of the diffraction pattern is 2 cm from the central peak, what is the wavelength of the incident light?

500 nm

Light incident on a 0.15 mm wide slit forms a diffraction pattern on a screen 6 m away. If the first minimum of the diffraction pattern is 2 cm from the central peak, what is the wavelength of the incident light? Answers: 500 nm 550 nm 420 nm 350 nm 633 nm

500 nm

A beam of green light is diffracted by a slit of width 0.55 mm. The diffraction pattern forms on a wall 2.06 m beyond the slit. The distance between the positions of zero intensity on both sides of the central bright fringe is 4.1 mm. What is the wavelength of the laser light in nm?

547

A beam of green light is diffracted by a slit of width 0.55 mm. The diffraction pattern forms on a wall 2.06 m beyond the slit. The distance between the positions of zero intensity on both sides of the central bright fringe is 4.1 mm. What is the wavelength of the laser light in nm? Enter an integer. Image3.gif

547

What wavelength monochromatic source in the visible region (390 to 710 nm) can be used to constructively reflect off a coating on glass (n = 1.46), if the coating is 191 nm thick?

558 nm

Laser light (λ = 578.5 nm) is incident on a single slit. What is the maximum width of the slit (in nm) for which no diffraction minima are observed?

579

A hydrogen atom is in a state for which the principle quantum number is n = 3. How many possible such states are there for which the quantum number m = 0?

6

A hydrogen atom is in a state for which the principle quantum number is n = 3. How many possible such states are there for which the quantum number m = 0? 6 10 2 4 8

6

Laser light (λ = 635.2 nm) is incident on a single slit. What is the maximum width of the slit (in nm) for which no diffraction minima are observed?

635

A group of quantum states of the hydrogen atom has n = 5. A subgroup of this group has I = 3. How many values of m are possible for the states in this subgroup?

7

A group of quantum states of the hydrogen atom has n = 5. A subgroup of this group has l = 3. How many values of m are possible for the states in this subgroup? Answers: 1 3 5 6 7 10 11

7

If the orbital angular momentum is l = 7, what is the magnitude of the orbital angular momentum, and what is the largest value its projection can have along any axis in units of ħ?

7.48 7

A screen is placed 46 cm from a single slit which is illuminated with 568 nm light. If the distance from the central maximum to the first minimum of the diffraction pattern is 3.6 mm, how wide is the slit in micrometer?

73

If the orbital angular momentum quantum number l in a hydrogen atom were allowed to take values from 0 to n (instead of n - 1, then number of electrons in the n = 1 shell would be

8

A telescope with a 6 m diameter lens is in orbit around Earth. What is its maximum resolution for 400 nm light, i.e. what is the smallest angle θ(rad) between objects it just can resolve?

8.13*10-8

A radioactive substance has a mean-life τ of 16 years. Assume we start with 344 kg of the substance. How many kg of the substance will be left after 23 years?

81.7

A hydrogenic ion with Z = 25 is excited from its ground state to the state with n = 7. How much energy (in eV) must be absorbed by the ion?

8326.5

Each proton-proton cycle generates 26.7 MeV of energy. If 2 Watts are generated via by the proton-proton cycle, how many neutrinos are produced?

940,000,000,000

A Young'sdouble-slit interference experiment is performed with monochromatic light. The separation between the slits is 0.41 mm. The interference pattern on the screen 4.1 m away shows the first maximum 5.4 mm from the center of the pattern. What is the wavelength of the light in nm?

(.41 X 5.4)/ 4100 = 540nm

A beam of green light is diffracted by a slit of width 0.55 mm. The diffraction pattern forms on a wall 2.06 m beyond the slit. The distance between the positions of zero intensity on both sides of the central bright fringe is 4.1 mm. What is the wavelength of the laser light in nm? Enter an integer.

(0.55 * 10-3)*(2.05*10-3 m / 2.06 m) =547nm

Electrons are ejected from a metal surface with energies ranging up to 0.5 eV when light with a wavelength of 525 nm is used. What is the work function of the metal in units of eV?

(1240/525)-0.5= 1.86

Which of the following are properties of electromagnetic (EM) waves? (1) EM waves have clearly identifiable position. (2) EM waves always travel in straight lines. (3) EM waves transport energy. (4) EM waves produce interference patterns. Answers: (2), (3), and (4) only All properties listed are properties of waves. (3) and (4) only. (4) only. (2) and (4) only.

(3) and (4) only.

What is the deBroglie wavelength (in nm) of a neutron (m = 1.67*10-27 kg) moving with a speed of 37 m/s? Enter a number with 2 decimal places.

(6.625*10^-34)/(1.67*10^-27*37)= 1.07*10^-8

Which of the two series of pictures has the diffraction pattern correctly paired with the corresponding slits? (Oct 29th lecture)

A

Why is this common picture of a neutral atom wrong? A. The relative scale is wrong. B. The pictured orbits violate the uncertainty principle. C. No matter how you look at it, this atom is not neutral.

A and B only

Which of the following statements are true statements about the electric field? (a) The strength of the electric field created by an object A at a point P depends on the separation between A and P. (b) The strength of the electric field created by an object A at a point P depends on the net charge of object A. (c) The strength of the electric field created by an object A at a point P depends on the charge of the test object used to measure it. (d) The strength of the electric field created by an object A at a point P is the magnitude of the electric force per unit charge A exerts on a test charge placed at P. Answers: (b), (c), (d) only (a), (b), (d) only (a), (b), (c) only (a) and (b) only All statements are true.

(a), (b), (d) only

What are the changes in the mass number A and the atomic number Z of a nucleus that undergoes β+ decay?

A is unchanged, Z decreases by 1.

What are the changes in the mass number A and the atomic number Z of a nucleus that undergoes β+ decay? Answers: a. A decreases by 2, Z increases by 1. b. A is unchanged, Z decreases by 1. c. A is unchanged, Z increases by 1. d. A decreases by 4, Z decreases by 2. e. A is unchanged, Z decreases by 2.

A is unchanged, Z decreases by 1.

Consider a particle tunneling through a barrier. Which of the following will yield the maximum increase in the likelihood of tunneling? (A) decrease the height of the barrier (B) increase the width of the barrier (C) increase the mass of the particle

A only

Assume that the potential V is higher at point A than at point B. Which statement then is correct? Answers: A positive charge gains electrostatic potential energy when it moves from point B to point A. Any charge can move along an equipotential line from point A to point B and neither gain or lose electrostatic potential energy. An externall force has to do positive work to move a positive charge from point A to point B. A negative charge gains electrostatic potential energy when it moves from point B to point A.

A positive charge gains electrostatic potential energy when it moves from point B to point A.

Which of the following is true?

After a measurement we can know the exact energy of a system. But there will be a spread in outcomes if we subsequently measure the position.

Which of the following statements about interference in a Young's double slit experiments is true? Answers: The double slit experiment provides evidence that light exhibits particle-like behavior. The double-slit experiment depends upon the use of white light from two sources. The two sources of light in the double-slit experiment could be two different light bulbs. For the equation for the interference maxima d sinθ = mλ to be geometrically valid, the wavelength of the light must be much, much greater than the slit separation distance. All the above statements are false.

All the above statements are false.

At what rate must Uranium 235 undergo fission by neutron bombardment to generate energy at a rate of 100 W (1 W = 1 J/s)? Assume each fission reaction releases 200 MeV of energy. Answers: Approximately 3*1014 fissions per second. Approximately 3*1012 fissions per second. Approximately 4*1011 fissions per second. Approximately 6*1012 fissions per second. Approximately 5*1010 fissions per second.

Approximately 3*1012 fissions per second.

At what rate must Uranium 235 undergo fission by neutron bombardment to generate energy at a rate of 100 W (1 W = 1 J/s)? Assume each fission reaction releases 200 MeV of energy.

Approximately 3x1012 fissions per second

A laser beam is used to read the information on an optical disc. In order to get more information on the disc, the wavelength of the laser used to read the information should be A. increased. B. decreased. C. The wavelength of the laser reading has nothing to do with how much information can be read.

B. decreased.

Radionuclides used in PET scanning are typically isotopes with short half-lives, for example carbon-11 (~20 min). Carbon-11 decays, and a Boron-11 nucleus is left behind. What type of decay is this?

Beta plus decay

Which of these elements is not produced by thermonuclear fusion processes in stellar interiors?

Bromine

Which of these elements is not produced by thermonuclear fusion processes in stellar interiors? Answers: Oxygen Silicon Carbon Bromine Chromium

Bromine

By what factor is the radius of the nucleus 160Dy bigger than that of 20Ne?

By a factor of 2

By what factor is the radius of the nucleus 160Dy bigger than that of 20Ne? Answers: By a factor of 8. By a factor of 4. By a factor of less than 1. The strength of the nuclear force increases and nuclei get smaller as more nucleons are added. By a factor of 1. All nuclei are about the same size. By a factor of 2.

By a factor of 2.

Assume that you observe a double-slit interference pattern with many fringes on a screen. The width of one of the slits is then halved, while everything else in the setup stays the same. Do you still observe the interference pattern on the screen? A. Yes, the pattern looks exactly the same. B. Yes, but the contrast between the maxima and minima increases. C. Yes, but the contrast between the maxima and minima decreases. D. No, there can be no interference pattern unless the slits have equal width. E. No, the diffraction pattern of the wider slit is now to narrow.

C. Yes, but the contrast between the maxima and minima decreases.

What is the first element in the periodic table to have a filled 4s subshell?

Ca

Which of the statements below is true?

Consider the photoelectric effect. The greater the work function of the target, the greater is the cutoff frequency.

Which of the statements below is true? a. Consider the photoelectric effect. The greater the work function of the target, the greater is the cutoff frequency. b. Consider the photoelectric effect. The greater the intensity of the incoming light, the smaller is the cutoff frequency. c. Photons can never be destroyed. d. Consider the photoelectric effect. The shorter the wavelength of the incoming light, the smaller is the cutoff frequency. e. Photons from a certain source have energy of 0.62 eV. This light is visible.

Consider the photoelectric effect. The greater the work function of the target, the greater is the cutoff frequency.

In the Compton effect, a photon of wavelength λ and frequency f hits an electron that is initially at rest. Which one of the following occurs as a result of the collision? A. The photon is absorbed completely. B. The photons gains energy, so the final photon has a wavelength < λ. C. The photons gains energy, so the final photon has a wavelength > λ . D. The photons loses energy, so the final photon has a wavelength < λ. E. The photons loses energy, so the final photon has a wavelength > λ.

E. The photons loses energy, so the final photon has a wavelength > λ.

Through rubbing an object has acquired a charge of -1 nanoCoulomb

Electrons have been transferred to the object

**Electrons are ejected from a metal surface with energies ranging up to 0.5 eV when light with a wavelength of 395 nm is used. What is the work function of the metal in units of eV? Enter a number with 2 decimal places.

Energy of photon = hc/lambda = 1240 eV nm /395 nm = 3.14 eV Work function = energy function = energy of photon - energy of electron = 3.14 - 0.5 = 2.64 eV Answer

What is the wavelength (in nm) of a photon with energy E = 3.0 eV

Energy of the photon , E = 3.0 eV = 3.0 eV * (1.6 *10^-19 J / 1 eV) E = 4.8 *10^-19 J The energy E of a photon is given by E = h c / λ (4.8 *10^-19 J) = ( 6.62 *10^-34 Js) (3 *10^8 m/s) / λ λ = 414 nm

Which of the statements below is correct for a photon.

Given the exact momentum of a photon, you know nothing about its position.

Consider a photon hitting the screen to the left of center as in the image. Which slit did it go through? IMAGE: PRACTICE 1 Answers: Neither Left Impossible to know. Determining the slit it went through and determining its trajectory and arrival position are incompatible measurements. Right Both

Impossible to know. Determining the slit it went through and determining its trajectory and arrival position are incompatible measurements.

The blue tint of a coated camera lens is largely caused by what effect?

Interference

What is the significance of the quantum number m when specifying energy eigenfunction or orbital of atoms?

It indicates the orientation of the orbital.

What is the significance of the quantum number m when specifying energy eigenfunction or orbital of atoms? Answers: a. It denotes the size of an eigenfunction or orbital. b. It represents the shape of the orbital. c. It indicates the orientation of the orbital. d. It represents the magnitude of the angular momentum of the orbital. e. None of the above. It just denotes the degeneracy.

It indicates the orientation of the orbital.

What is the definition of the of nuclear binding energy?

It is the amount of energy needed to completely separate a nucleus into its component neutrons and protons.

What is the definition of the nuclear binding energy?

It is the amount of energy needed to completely separate a nucleus into its component neutrons and protons..

What is the definition of the nuclear binding energy? Answers: a. It is the amount of energy needed to remove the most loosely bound electron from an atom. b. It is the amount of energy needed to remove all electrons from an atom.. c. It is the amount of energy needed to completely separate a nucleus into its component neutrons and protons.. d. It is the amount of energy needed to remove all electrons from an atom and then take the nucleus apart. e. It is the amount of energy needed to remove the most loosely bound proton from a nucleus.

It is the amount of energy needed to completely separate a nucleus into its component neutrons and protons..

The configuration of the potassium atom is 1s2 2s2 2p6 3s2 3p6 4s1. Which of the following statements about potassium is true?

Its electron charge distribution or probability density is spherically symmetric.

The configuration of the potassium atom is 1s2 2s2 2p6 3s2 3p6 4s1. Which of the following statements about potassium is true? Answers: a. Its electron charge distribution or probability density is spherically symmetric. b. Its least tightly bound electron has l = 4. c. Its n = 3 shell is completely filled. d. Its atomic number is 17. e. Its 4s subshell is completely filled.

Its electron charge distribution or probability density is spherically symmetric.

Two stars that are close together are photographed through a telescope. Which situation would result in the most clearly separated images of the stars?

Large lens, blue stars

Choose the correct statement. 1. Nearly the entire mass of the atom is concentrated in the nucleus and the mass of the electrons is almost negligible. 2. Nearly the entire charge of the atom is concentrated in the nucleus and the charge of the electrons is almost negligible. 3. STM images reveal the distribution of both the negative and the positive charge in an atom. 4. Mass is uniformly distributed throughout the atom and hence most alpha-particles are deflected in the backward direction in alpha - particle scattering experiments.

Nearly the entire mass of the atom is concentrated in the nucleus and the mass of the electrons is almost negligible.

A coil is connected to a galvanometer, which can measure the current flowing through the coil. You are not allowed to connect a battery to this coil. Given a magnet, a battery and a long piece of wire, can you induce a steady current in that coil? Answers: Yes, move the magnet back and forth through the coil. No, it is not possible to induce a steady current in the coil. Yes, connect the two ends of the wire to the battery and stretch the wire out next to the coil. Yes, just set the magnet inside the coil.

No, it is not possible to induce a steady current in the coil.

**Each proton-proton cycle generates 26.7 MeV of energy. If 2.6 Watts are generated via by the proton-proton cycle, how many neutrinos are produced?

Number of proton-proton cycles required to generate E energy is N = E / E ' Where E ' = Energy generated in proton-proton cycle = 26.7 MeV = 26.7 x1.6 x10 -13 J Substitute values you get N = 2.6 J /(26.7 x1.6 x10 -13 J ) = 6.086 x10 17 Therefore number of neutrions produced = 2 x Number of proton-proton cycles = 2N = 1.217 x10 18

In the nuclear reaction given by 147N + 42He = nmX + 11H, what is the nucleus nmX? Answers: Nitrogen of mass number 17 Oxygen of mass number 18 Nitrogen of mass number 18 Oxygen of mass number 17 Oxygen of mass number 16

Oxygen of mass number 17

To sustain laser action it is usually necessary to place the lasing material between the two mirrors of an optical cavity. Why?

Photons are reflected back and forth through the lasing medium, which greatly increases the probability of stimulated emission.

To sustain laser action it is usually necessary to place the lasing material between the two mirrors of an optical cavity. Why? Answers: a. The Pauli exclusion principle requires that the photons have different quantum numbers. The cavity guaranties this. b. Photons are reflected back and forth through the lasing medium, which greatly increases the probability of stimulated emission. c. We have to confine the photons, because confinement leads to energy quantization. d. The mirrors produce thin-film interference, which enhances laser action. e. This is required by the uncertainty principle.

Photons are reflected back and forth through the lasing medium, which greatly increases the probability of stimulated emission.

**At what rate must Uranium 235 fission by neutron bombardment to generate energy at the rate of 915 W (1W=1J/s)? Assume each fission reaction releases 200 MeV of energy. How many billions of fissions per second are required? Use 1 eV = 1.6 * 10^19 J.

Rate at which energy is generated = 915 W =915J/s Each fission releases 200MeV of energy Number of fissions per second = (915)/(200×10^(6)×1.6×10^(-19)) Number of fission per second = 28593.75 × 10^(9) Approximately 28594 ×10^(9) fissions per second occur Therefore 28594 billions of fissions per second occur

Compare the speed of red light and blue light in free space. Answers: Blue light travels faster. Red light travels faster. Red and blue light travel at the same speed. The speed of the light depends on the relative motion of the source and the observer. Since we have no information about this relative motion, we cannot give a definite answer.

Red and blue light travel at the same speed.

Which of the following statements is false of total internal reflection (TIR)? Answers: When TIR occurs, the reflected light obeys the law of reflection. If TIR occurs at the boundary of water and air, then the light must be within water and heading towards the boundary with air. TIR causes a portion of the light to be refracted and the rest to be reflected. TIR only occurs when the angle of incidence is greater than the critical angle. All the above statements are true.

TIR causes a portion of the light to be refracted and the rest to be reflected.

The long-range electrostatic repulsion between protons limits the size of stable nuclei. Why are there no large nuclei consisting only of neutrons, which do not repel each other?

The Pauli exclusion principle would require the neutrons to occupy very high energy states, yielding the nucleus unstable.

To sustain laser action it is usually necessary to place the lasing material between the two mirrors of an optical cavity. Why?

The cavity recycles most of the photons and therefore increases the probability of duplication.

**Laser light (λ = 546 nm) is incident on a single slit. What is the maximum width of the slit (in nm) for which no diffraction minima are observed? Round to the closest integer.

The diffraction minimas are defined like d sinθ = m λ if θ = 90 ° there is no minima observed, so d sin(90) = m λ d = m λ if m = 1 d = λ d = 546 nm The slit can not be smaller that the wavelength of the light

**A hydrogenic ion with Z = 16 is excited from its ground state to the state with n = 7. How much energy (in eV) must be absorbed by the ion? Enter a number with one decimal place after the decimal point.

The energy of each electron energy level is E = -13.6 eV/n^2 The absorbed energy is, E = -13.6 eV(1/16^2 - 1/1^2) = 13.54 eV

How does the total mass of the fission fragments compare to the mass of the original nucleus in a fission reaction?

The fission fragments have less mass than the original nucleus.

Violet light is able to eject electrons from the surface of potassium metal, whereas red light cannot. Why?

The frequency of red light is lower than that of violet light and its photon energy is not sufficient to eject electrons.

Violet light is able to eject electrons from the surface of potassium metal, whereas red light cannot. Why? a. The intensity of violet light is always greater than that of red light. b. The frequency of red light is lower than that of violet light and its photon energy is not sufficient to eject electrons. c. The wavelength of violet light is greater than that of red light and its energy is therefore spread over more electrons, giving it more targets. d. Red light is more efficiently absorbed by air and therefore does not reach the metal. e. None of the above.

The frequency of red light is lower than that of violet light and its photon energy is not sufficient to eject electrons.

Consider a double slit interference experiment with electrons. What happens to the interference pattern if we increase the momentum of the electrons?

The fringes get closer together.

You observe that a glass prism separates white light into its component colors. Which one of the following statements best explains this observation? a) Some of the color components of the white light are absorbed by the glass and only the remaining components are observed. b) Only some of the color components are refracted by the glass; and these are the ones that are observed. c) White light is separated into its color components by total internal reflection within the prism. d) The separation of white light into its color components is due to the increase in the speed of light within the glass. e) The index of refraction of the glass depends on the wavelength, so the color components are refracted at different angles.

The index of refraction of the glass depends on the wavelength, so the color components are refracted at different angles.

The figure below shows a red line and a green line of the same order in the pattern produced by a diffraction grating. If we increased the grating spacing d, would the lines shift to the right, shift to the left, or remain in place.

The lines would shift to the right.

The figure below shows a red line and a green line of the same order in the pattern produced by a diffraction grating. If we increased the grating spacing d, would the lines shift to the right, shift to the left, or remain in place. Answers: a. The red line would shift towards the right and the green line towards the left. b. The lines would remain in place. c. The lines would shift to the right. d. The lines would shift to the left. e. The red line would shift towards the left and the green line towards the right.

The lines would shift to the right.

Which of the following is not true?

The nuclear force and the electric force between two protons close to each other have approximately the same magnitude.

In the Compton effect, a photon of wavelength λ and frequency f hits an electron that is initially at rest. Which one of the following occurs as a result of the collision?

The photons loses energy, so the final photon has a wavelength > λ.

How does the binding energy per nucleon of a fusion product compare to that of the pieces that combined to form it?

The product has a greater binding energy per nucleon than the pieces.

Why does a soap bubble reflect virtually no light just before it bursts?

The reflected light from the front and back surface destructively interfere, because the path difference is nearly zero, but the phase shift upon reflection differs by 180o for the two surfaces.

Two lamps, one with a thick filament and one with a thin filament, are connected in series. Answers: More current flows through the lamp with the thin filament. The lamp with the thick filament will appear brighter. More current flows through the lamp with the thick filament. The same current flows through both filaments.

The same current flows through both filaments.

For the electron in the hydrogen atom, which of the sets of observables below are compatible observables?

The total energy, the magnitude of the orbital angular momentum and its component along the z-axis.

For the electron in the hydrogen atom, which of the sets of observables below are compatible observables? Answers: a. The x, y, and z component of the orbital angular momentum. b. No compatible observables exist. The uncertainty principle prevents us from knowing the value of two observables simultaneously. c. The total energy, the magnitude of the orbital angular momentum and its component along the z-axis. d. The momentum, the magnitude of the orbital angular momentum and its component along the z-axis. e. The position, the total energy and the magnitude of the orbital angular momentum.

The total energy, the magnitude of the orbital angular momentum and its component along the z-axis.

On a balance scale, you put 2 neutrons and 1 proton on one side and you put a tritium nucleus (3H) on the other. Which side weighs more?

The two neutrons and 1 proton.

14C is produced in the upper atmosphere by cosmic rays. Living organisms uptake 14C. Assume we isolate the same quantity of 14C from a 1000 year old fossil and from a recently harvested tomato. We monitor the decay of these quantities.

The two samples will have approximately the same number of decays per second.

14C is produced in the upper atmosphere by cosmic rays. Living organisms uptake 14C. Assume we isolate the same quantity of 14C from a 1000 year old fossil and from a recently harvested tomato. We monitor the decay of these quantities. Answers: a. The carbon isolated from the fossil will have more decays per second that the carbon isolated from the tomato plant. b. The two samples will have approximately the same number of decays per second. c. The carbon isolated from the tomato plant will have more decays per second that the carbon isolated from the fossil. d. It is impossible to compare the decay rates, since nuclear decay is a random process.

The two samples will have approximately the same number of decays per second.

Does the wave function let us make predictions about the outcome of measurements?

The wave function lets us make predictions about the outcome of all measurements, but we have to operate on it with an operator corresponding to the measurement we want to make.

Does the wave function let us make predictions about the outcome of measurements? a. The wave function lets us make predictions about the outcome of all measurements, but we have to operate on it with an operator corresponding to the measurement we want to make. b. The wave function only lets us make predictions about the outcome of position measurements. c. The wave function is just a mathematical construct. It contains no information about the outcome of real measurements. d. The wave function only lets us make predictions about the outcome of energy measurements. e. The wave function tells us that the outcome of all measurements is uncertain.

The wave function lets us make predictions about the outcome of all measurements, but we have to operate on it with an operator corresponding to the measurement we want to make.

**A gamma ray photon has an energy of 0.55 GeV. (1 GeV = 109 eV.) What is the wavelength of the gamma ray in fm? (1 nm = 10-15 m)?

The wavelength of gamma ray in fm will be given as : using a formula, we have E = h f = h c / (lamda) (lamda) = h c / E where, E = photon energy = 0.55 GeV = 8.81197 x 10-11 J then, we get (lamda) = (6.62 x 10-34 J.s) (3 x 108 m/s) / (8.81197 x 10-11 J) (lamda) = 2.25 x 10-15 m (lamda) = 2.25 fm

The figure shows two single-slit diffraction patterns. The distance between the slit and the viewing screen is the same in both cases. Which of the following could be true?

The wavelengths are the same for both. The width of slit 1 is larger than the width of slit 2.

The figure shows two single-slit diffraction patterns. The distance between the slit and the viewing screen is the same in both cases. Which of the following could be true? Answers: a. The slit widths and the wavelengths are the same for both. The intensity of the light falling onto slit 1 is smaller than the intensity falling onto slit 2. b. The wavelengths are the same for both. The width of slit 1 is larger than the width of slit 2. c. The slit widths and the wavelengths are the same for both. The intensity of the light falling onto slit 1 is larger than the intensity falling onto slit 2. d. The slit widths are the same but the wavelength incident on slit 1 is larger than the wavelength incident on slit 2. e. The wavelengths are the same for both. The width of slit 1 is smaller than the width of slit 2.

The wavelengths are the same for both. The width of slit 1 is larger than the width of slit 2.

Which of the statements below is true? Answers: Diffraction suggests that light is a particle, and interference suggests that light is a wave. There would be no double slit interference pattern, if there were no single slit diffraction. The meeting of a trough of one wave with a trough of another wave results in destructive interference. In a double slit experiment, the interference pattern disappears if you block one of the slits, and you will just observe the single slit diffraction pattern. This shows that light behaves like a particle. All the above statements are true. All the above statements are false.

There would be no double slit interference pattern, if there were no single slit diffraction.

Assume that in the experiment shown below electrons are ejected with some kinetic energy when violet light hits a metal surface. Which of the following changes to the experiment would decrease the maximum kinetic energy of the ejected electrons.

Using blue light

Which is correct language usage? Answers: It's just semantics; it doesn't matter. Voltage in a resistor, current through a resistor Voltage against a resistor, current across a resistor Voltage through a resistor, current in a resistor Voltage across a resistor, current through a resistor

Voltage across a resistor, current through a resistor

Assume that you observe a double-slit interference pattern with many fringes on a screen. The width of one of the slits is then halved, while everything else in the setup stays the same. Do you still observe the interference pattern on the screen?

Yes, but the contrast between the maxima and minima decreases.

After students have completed an entire lab on polarization, the instructor asks them for a sketch of a polarized light wave. One student provides the sketch shown below. Does this sketch represent a polarized wave? IMAGE REVIEW PROBLEMS Answers: No, the magnetic field is missing. No, it is just a plane old wave. No, the student did not draw a polarizer, which is needed to define the polarization. Yes, it is a transverse wave and the electric field has a well-defined direction.

Yes, it is a transverse wave and the electric field has a well-defined direction.

Can you take a picture of a virtual image with a camera? Answers: No, there is no real source or object. Yes, the camera just produces a virtual picture. No, the camera lens produces only real images at the sensor or film location. Yes, the camera lens focuses light from a virtual image in exactly the same way as light coming directly from an object

Yes, the camera lens focuses light from a virtual image in exactly the same way as light coming directly from an object

Which of the following statements are true about the electric potential or the electric potential difference? (a) The SI unit of the electric potential is volts (V). (b) The electric potential can also be expressed in units of Joules. (c) The electric potential produced by a charge at a given location provides a measure of the rate at which charge flows past that point. (d) As a positive charge moves in the direction of an electric field, it gains electric potential. (f) The electric potential difference between two points is the difference in potential energy possessed by another charge at those two points, divided by this charge

a & f only

In a Compton scattering experiment, a beam of x-rays of wavelength 0.07 nm scatters of nearly free electrons. The scattered radiation is investigated as a function of the angle theta between it and the primary beam. The scattered radiation contains

a component shifted in wavelength by an amount that is independent of the wavelength of the primary beam.

In a Compton scattering experiment, a beam of x-rays of wavelength 0.07 nm scatters of nearly free electrons. The scattered radiation is investigated as a function of the angle theta between it and the primary beam. The scattered radiation contains Answers: no wavelength other than that of the primary beam. a component shifted in wavelength by an amount that is independent of the wavelength of the primary beam. a component shifted in wavelength by an amount that decreases as the scattering angle increases. two components with wavelength shifted up and down by an equal amount. a component shifted in wavelength by an amount that is independent of the scattering angle.

a component shifted in wavelength by an amount that is independent of the wavelength of the primary beam.

The total energy of a particle is the sum of its kinetic energy and its potential energy. The zero of the potential energy can be chosen arbitrarily, so the potential energy can be negative. Classically forbidden regions are where

a particle's total energy is less than its potential energy

The total energy of a particle is the sum of its kinetic energy and its potential energy. The zero of the potential energy can be chosen arbitrarily, so the potential energy can be negative. Classically forbidden regions are where a. a particle's total energy is less than its potential energy b. a particle's total energy is less than its kinetic energy. c. a particle's total energy is greater than its kinetic energy d. a particle's total energy is greater than its potential energy e. a particle's total energy is equal its potential energy

a particle's total energy is less than its potential energy

The isotope 234 92U undergoes five successive alpha decays. Identify the correct decay chain a) 230 90Th --> 226 88Ra --> 222 86Rn --> 218 84Po --> 214 82Pb b) 232 90Th --> 228 88Ra --> 228 86Rn --> 220 84Po --> 216 82Pb c) 228 90Th --> 228 88Ra --> 228 86Rn --> 218 84Po --> 212 82Pb d) 233 90Th --> 229 88Ra --> 225 86Rn --> 218 84Po --> 217 82Pb

a) 230 90Th --> 226 88Ra --> 222 86Rn --> 218 84Po --> 214 82Pb

Suppose a Uranium 238 nucleus "swallows" a neutron and then decays not by fission but by beta decay, emitting an electron and an anti-neutrino. Which nucleus remains after this decay? a) 239Np b) 239U c) 238Pa d) 238Np e) 239Pu

a) 239Np

What are the changes in the mass number A and the atomic number Z of a nucleus that undergoes β+ decay? a) A is unchanged, Z decreases by 1. b) A decreases by 4, Z decreases by 2. c) A is unchanged, Z decreases by 2. d) A decreases by 2, Z increases by 1. e) A is unchanged, Z increases by 1.

a) A is unchanged, Z decreases by 1.

Which of the following statements is true? You can choose more than 1. a) As light travels from vacuum (n = 1) into glass (n > 1), the wavelength of the light changes. b) As light travels from vacuum (n = 1) into glass (n > 1), the speed of the light changes. c) An image that is formed behind a mirror is always virtual. d) An image that is formed behind a mirror is always inverted. e) As light travels from vacuum (n = 1) into glass (n > 1), the frequency of the light changes.

a) As light travels from vacuum (n = 1) into glass (n > 1), the wavelength of the light changes. b) As light travels from vacuum (n = 1) into glass (n > 1), the speed of the light changes. c) An image that is formed behind a mirror is always virtual.

What is the significance of the quantum number m when specifying energy eigenfunction or orbital of atoms? a) It indicates the orientation of the orbital. b) It denotes the size of an eigenfunction or orbital. c) None of the above. It just denotes the degeneracy. d) It represents the shape of the orbital. e) It represents the magnitude of the angular momentum of the orbital.

a) It indicates the orientation of the orbital

To sustain laser action it is usually necessary to place the lasing material between the two mirrors of an optical cavity. Why? a) Photons are reflected back and forth through the lasing medium, which greatly increases the probability of stimulated emission. b) The Pauli exclusion principle requires that the photons have different quantum numbers. The cavity guaranties this. c) This is required by the uncertainty principle. d) We have to confine the photons, because confinement leads to energy quantization. e) The mirrors produce thin-film interference, which enhances laser action

a) Photons are reflected back and forth through the lasing medium, which greatly increases the probability of stimulated emission.

Two light bulbs, one rated at 50 W and a second rated at 100 W, are both supposed to be connected to a 110 V source of emf. Which one of the following is true? a) The 50 W bulb has twice the resistance as the 100 W bulb. b) The 50 W bulb has one quarter the resistance as the 100 W bulb. c) Both bulbs have the same resistance, since they are connected to the same 110 V emf. d) The 50 W bulb has half the resistance as the 100 W bulb. e) The 50 W bulb has four times the resistance as the 100 W bulb

a) The 50 W bulb has twice the resistance as the 100 W bulb.

In a Compton scattering experiment, a beam of x-rays of wavelength 0.07 nm scatters of nearly free electrons. The scattered radiation is investigated as a function of the angle theta between it and the primary beam. The scattered radiation contains a) a component shifted in wavelength by an amount that is independent of the wavelength of the primary beam. b) no wavelength other than that of the primary beam. c) a component shifted in wavelength by an amount that is independent of the scattering angle. d) two components with wavelength shifted up and down by an equal amount. e) a component shifted in wavelength by an amount that decreases as the scattering angle increases.

a) a component shifted in wavelength by an amount that is independent of the wavelength of the primary beam.

A person who is "near-sighted", or myopic, cannot focus on faraway objects (objects at infinity) because the curvature of the person's eye-lens is too great. This causes parallel rays (from a distant point source) to bend too much and focus in front of the retina. The person sees a fuzzy patch of light rather than a sharp point. This person needs eyeglasses with lenses that are a) diverging. b) either diverging or converging, depending on the prescription. c) converging. d) polarized. e) corrected for astigmatism.

a) diverging.

A current in a long, straight wire produces a magnetic field. The magnetic field lines a) form circles that go around the wire. b) are parallel to the wire. c) come in from infinity to the wire. d) go out from the wire to infinity. e) form circles that pass through the wire

a) form circles that go around the wire.

A paraxial ray a) is a ray that moves nearly parallel to the optical axis. b) is a ray that has been reflected from parabolic mirror. c) is a ray that moves exactly parallel to the optical axis. d) moves in a parabolic path

a) is a ray that moves nearly parallel to the optical axis.

At a boundary between two transparent media, light may be refracted. The angle of incidence a) may be greater than, less than, or equal to the angle of refraction. b) is always less than the angle of refraction. c) must equal the angle of refraction. d) is always greater than the angle of refraction.

a) may be greater than, less than, or equal to the angle of refraction.

The direction of the magnetic field in an electromagnetic wave is a) perpendicular to the electric field. b) anti-parallel to the electric field. c) parallel to the direction of propagation of the wave. d) parallel to the electric field.

a) perpendicular to the electric field.

The nuclear force is a) short range and charge independent. b) electromagnetic in nature. c) short range and charge dependent. d) long range and charge independent e) long range and charge dependent

a) short range and charge independent.

Why does a soap bubble reflect virtually no light just before it bursts? Answers: a. The reflected light from the front and back surface destructively interfere, because the path difference is nearly zero, but the phase shift upon reflection differs by 180o for the two surfaces. b. It absorbs all the light energy, so that it has enough energy to burst. c. This is an example of polarization by reflection. The light becomes p-polarized, but only s-polarized light is reflected. d. We have total internal reflection. The light keeps on reflecting inside the bubble until the bubble bursts. e. Light does not interact with a soap film.

a. The reflected light from the front and back surface destructively interfere, because the path difference is nearly zero, but the phase shift upon reflection differs by 180o for the two surfaces.

Which one is the most massive of the particles listed below? Answers: gamma ray alpha particle neutron electron proton

alpha particle

Consider both the nuclear and the electrostatic interaction. A proton in a very large nucleus

attracts some protons and repels others.

Three resistors connected in series carry currents labeled I1, I2, I3, respectively. Which of the following expresses the value of the total current IT in the system made up of the three resistors? a) IT = (1/I1 + 1/I2 + 1/I3) b) IT = I1 = I2 = I3 c) IT = 3I1 = 3I2 = 3I3 d) IT = I1 + I2 + I3 e) IT = (1/I1 + 1/I2 + 1/I3)-1

b) IT = I1 = I2 = I3

Violet light is able to eject electrons from the surface of potassium metal, whereas red light cannot. Why? a) The intensity of violet light is always greater than that of red light. b) The frequency of red light is lower than that of violet light and its photon energy is not sufficient to eject electrons. c) None of the above. d) Red light is more efficiently absorbed by air and therefore does not reach the metal. e) The wavelength of violet light is greater than that of red light and its energy is therefore spread over more electrons, giving it more targets.

b) The frequency of red light is lower than that of violet light and its photon energy is not sufficient to eject electrons.

For the electron in the hydrogen atom, which of the sets of observables below are compatible observables? a) The x, y, and z component of the orbital angular momentum. b) The total energy, the magnitude of the orbital angular momentum and its component along the z-axis. c) The position, the total energy and the magnitude of the orbital angular momentum. d) No compatible observables exist. The uncertainty principle prevents us from knowing the value of two observables simultaneously. e) The momentum, the magnitude of the orbital angular momentum and its component along the z-axis

b) The total energy, the magnitude of the orbital angular momentum and its component along the z-axis.

Does the wave function let us make predictions about the outcome of measurements? a) The wave function only lets us make predictions about the outcome of energy measurements. b) The wave function lets us make predictions about the outcome of all measurements, but we have to operate on it with an operator corresponding to the measurement we want to make. c) The wave function tells us that the outcome of all measurements is uncertain. d) The wave function is just a mathematical construct. It contains no information about the outcome of real measurements. e) he wave function only lets us make predictions about the outcome of position measurements.

b) The wave function lets us make predictions about the outcome of all measurements, but we have to operate on it with an operator corresponding to the measurement we want to make.

Heisenberg's Uncertainty Principle of quantum mechanics a) is a mathematical formula that lets us calculate the momentum of an electron or other particle if we know its position. b) does none of the above. c) expresses a relationship between the wavelength and frequency of a photon or other particle. d) is a mathematical formula that lets us calculate the velocity of an electron or other particle if we know its energy. e) relates the energy and frequency of an electron or other particle.

b) does none of the above.

Rays that pass through a lens very close to the principle axis are more sharply focused than those that are very far from the axis. This spherical aberration helps us understand why a) we become more near-sighted as we become older. b) it is easier to read in bright light than it is in dim light. c) we become more far-sighted as we become older. d) we can only see in black and white in dim light.

b) it is easier to read in bright light than it is in dim light.

It is necessary to coat a glass lens with a nonreflecting layer. If the wavelength of the light in the coating is λ, the best choice is a layer of material having an index of refraction between those of glass and air and a thickness of a) 1.5 λ b) λ/4 c) λ d) λ/√(2)

b) λ/4

We observe an interference patterns using light. This suggests that light

behaves like a wave.

Diffraction is the tendency of light to a) bend around corners and edges. b) separate into its component colors. c) reflect off a mirror. d) disperse within a prism. e) bend through a lens.

bend around corners and edges

Diffraction is the tendency of light to

bend around corners and edges.

Diffraction is the tendency of light to Answers: a. bend around corners and edges. b. disperse within a prism. c. reflect off a mirror. d. bend through a lens. e. separate into its component colors.

bend around corners and edges.

The cesium isotope 137Cs decays into the barium isotope 137Ba with a half-life of 30.2 years? The atomic masses of cesium and barium are 136.9071 u and 136.9058 u. The decay is a

beta minus decay. An electron is emitted.

The cesium isotope 137Cs decays into the barium isotope 137Ba with a half-life of 30.2 years? The atomic masses of cesium and barium are 136.9071 u and 136.9058 u. The decay is a Answers: a. beta plus decay. A positron is emitted. b. a chain reaction. c. a gamma decay. d. alpha decay. e. beta minus decay. An electron is emitted.

beta minus decay. An electron is emitted.

Light composed of red (700 nm) and blue (400 nm) components falls on two slits and forms a central magenta-appearing line on a screen 2 meters away. The nearest bright line to the central line is Answers: magenta. blue. red. white.

blue.

Light composed of red (700 nm) and blue (400 nm) components falls on two slits and forms a central magenta-appearing line on a screen 2 meters away. The nearest bright line to the central line is blue. Answers: white. red. magenta. blue.

blue.

Which of the following types of aberrations is not produced by spherical mirrors? a) Spherical aberration b) Astigmatism c) Chromatic abberation d) Coma e) Distortion

c) Chromatic abberation

A parallel combination of resistors has an equivalent (or effective) resistance of 2 Ω. Which one of the following is true? a) The sum of the individual resistances is 2 Ω. b) The sum of the reciprocals of the individual resistances is 2 Ω. c) Each of the individual resistances is greater than 2 Ω. d) Each of the individual resistances is smaller than 2 Ω. e) none of the above

c) Each of the individual resistances is greater than 2 Ω.

14C is produced in the upper atmosphere by cosmic rays. Living organisms uptake 14C. Assume we isolate the same quantity of 14C from a 1000 year old fossil and from a recently harvested tomato. We monitor the decay of these quantities. a) The carbon isolated from the tomato plant will have more decays per second that the carbon isolated from the fossil. b) It is impossible to compare the decay rates, since nuclear decay is a random process. c) The two samples will have approximately the same number of decays per second. d) The carbon isolated from the fossil will have more decays per second that the carbon isolated from the tomato plant.

c) The two samples will have approximately the same number of decays per second.

Which one of the following phrases most accurately describes the term wave front? a) a surface upon which a wave is incident b) the surface of a plane mirror c) a surface of constant phase within a wave d) a surface that is parallel to the direction of wave propagation e) the surface of a convex mirror

c) a surface of constant phase within a wave

The initial fragments formed by fission a) are only neutrons. b) have more protons than neutrons. c) have more neutrons than protons. d) have about the same number of neutrons and protons. e) are a daughter nucleus and a alpha particle.

c) have more neutrons than protons.

A beam of light A has twice the wavelength but the same intensity as a beam B. The number of photons that hit a given area in a given time, when illuminated by beam B is a) twice the number that hit when the area is illuminated by beam A. b) the same as the number that hit when the area is illuminated by beam A. c) one-half the number that hit when the area is illuminated by beam A. d) four times the number that hit when the area is illuminated by beam A. e) one-quarter the number that hit when the area is illuminated by beam A.

c) one-half the number that hit when the area is illuminated by beam A

An electromagnetic wave is transporting energy in the positive y direction. At one point and one instant the magnetic field is in the positive x direction. The electric field at that point and instant points in the a) positive y direction. b) negative z direction c) positive z direction d) negative y direction. e) negative x direction

c) positive z direction

The focal length of a converging lens is a) the distance at which an object must be placed to form an image. b) the distance from the front surface to the back surface. c) the distance at which parallel light rays are focused. d) the distance at which an image is formed.

c) the distance at which parallel light rays are focused.

A bird sits on a high-voltage line. Why does the bird not get hurt? a) The bird is very lucky. b) The bird is very small, and therefore can hold only a small amount of charge. c) The whole bird is at approximately the same potential, so there is no electric field across it. d) The bird's charge is opposite to that of the line, and the charges cancel out. e) Birds are not affected by electric fields

c) the whole bird is at approximately the same potential, so there is no electric field across it.

The interference pattern from two closely spaced slits can only is only produced if single slit diffraction spreads out the light. The graph below shows a double slit interference pattern. The ideal two-slit intensity pattern is multiplied by intensity of the single slit diffraction pattern. If this pattern was obtained with two slits spaced by d = 0.1 mm, what was the slit width w? a. w = d b. w = d/2 c. w = d/4 d. w = d/8 e. This cannot be determined from this graph.

c. w = d/4

It is necessary to coat a glass lens with a nonreflecting layer. If the wavelength of the light in the coating is λ, the best choice is a layer of material having an index of refraction between those of glass and air and a thickness of a. 1.5 λ b. λ/2 c. λ/4 d. λ e. λ/√(2)

c. λ/4

The radius of an atomic nucleus is of the order of a) The nucleus is a point and has no radius. b) 10^-8 m. c) 10^-18 m. d) 10^-15 m. e) 10^-10 m.

d) 10^-15 m.

Which of these elements is not produced by thermonuclear fusion processes in stellar interiors? a) Oxygen b) Silicon c) Carbon d) Bromine e) Chromium

d) Bromine

One of the ways in which a coin operated vending machine checks to make sure that the coins fed to it are genuine is to roll them past a strong magnet a) The coins will become magnetized and thus can easily be sorted. b) Only coins made of magnetic materials such as iron will be affected by the magnet. c) Coins made of a good conductor will speed up as they roll past the magnet. d) Coins made of a good conductor will slow down as they roll past the magnet.

d) Coins made of a good conductor will slow down as they roll past the magnet.

What is the definition of the nuclear binding energy? a) It is the amount of energy needed to remove all electrons from an atom.. b) It is the amount of energy needed to remove the most loosely bound proton from a nucleus. c) It is the amount of energy needed to remove all electrons from an atom and then take the nucleus apart. d) It is the amount of energy needed to completely separate a nucleus into its component neutrons and protons. e) It is the amount of energy needed to remove the most loosely bound electron from an atom.

d) It is the amount of energy needed to completely separate a nucleus into its component neutrons and protons.

The magnetic domains in a non-magnetized piece of iron are characterized by which orientation? a) Adjacent domains are aligned anti-parallel to each other. b) Adjacent domains are aligned at 45o with respect to each other. c) Adjacent domains are oriented perpendicular to each other. d) They are oriented randomly. e) They are all aligned parallel to each other.

d) They are oriented randomly.

Which of the following statements about a conductor in electrostatic equilibrium is false? a) A charge located within a hole in a conductor at equilibrium feels no force from charges outside the conductor. b) The electric field inside the conductor is zero. c) Just outside the conductor, the electrostatic field is perpendicular to its surface. d) All the above statements are true. e) The net charge inside the conductor is zero, all excess charge resides on the surface.

d) all are true

The cesium isotope 137Cs decays into the barium isotope 137Ba with a half-life of 30.2 years? The atomic masses of cesium and barium are 136.9071 u and 136.9058 u. The decay is a a) alpha decay. b) beta plus decay. A positron is emitted. c) a gamma decay. d) beta minus decay. An electron is emitted. e) a chain reaction.

d) beta minus decay. An electron is emitted

A material is known to be an isotope of lead, although the particular isotope is not known. From such limited information, which of the following quantities can you specify: its atomic number, its neutron number, its atomic mass number? a) its neutron number and its atomic mass number b) its atomic number and its atomic mass number c) only its atomic mass number d) only its atomic number e) only its neutron number

d) only its atomic number

Which image cannot be created by a converging lens? An image that is a) inverted and smaller than the object. b) real and larger than the object. c) virtual. d) right side up and smaller than the object. e) none of these

d) right side up and smaller than the object.

Bohr's model of the atom accounted for the spectra of a) many low atomic number atoms b) all noble gases. c) normal Helium atoms. d) single-electron atoms. e) all atoms

d) single-electron atoms.

A certain physics textbook shows a region of space in which two electric field lines cross each other. We should conclude that: a) an electrical conductor is present. b) the field points in two directions at the same place. c) an insulator is present. d) the author of the textbook made a mistake. e) at least two point charges are present.

d) the author of the textbook made a mistake

Which statement is correct? When light is incident on an interface, the angle(s) of a) the reflected and the refracted beam depend on the wavelength. b) the refracted beam is independent of the wavelength. c) the reflected and the refracted beam are independent of frequency. d) the reflected beam is independent of the wavelength.

d) the reflected beam is independent of the wavelength.

Which statement is false? Quantum mechanics predicts that a) it is impossible to know the exact energy of a particle and to be able to track the particle. b) confined particles can only have certain discrete energies. c) if, in principle, you can only observe a particle for a short time, you cannot know its energy exactly. d) you can never know the energy and the magnitude of the angular momentum exactly at the same time. e) all angular momentum is quantized in magnitude as well as in direction.

d) you can never know the energy and the magnitude of the angular momentum exactly at the same time.

An electron with an energy of 1000 eV has a deBroglie wavelength of 0.04 nm. If the speed of the electron increases by a factor of 2, the wavelength will

decrease by a factor of 1/2.

The magnitude of the momentum of a non-relativistic particle is p = mv. An electron with momentum p and energy E = 1000 eV has a de Broglie wavelength of 0.04 nm. If the speed of the electron increases by a factor of 10, the de Broglie wavelength will Answers: remain the same. increase by a factor of 10. decrease by a factor of 1/√10. decreased by a factor of 1/10 . increase by a factor of √10.

decreased by a factor of 1/10 .

A laser beam is used to read the information on an optical disc. In order to get more information on the disc, the wavelength of the laser used to read the information should be

decreased.

A beam of light that consists of a mixture of red, green and violet light strikes a prism (surrounded by air). Which physical phenomenon is illustrated by the fact that the prism has different refractive indices for different colors? Answers: dispersion diffraction reflection Doppler effect interference

dispersion

Consider a particle tunneling through a barrier. Which of the following will yield the maximum increase in the likelihood of tunneling? (A) decrease the height of the barrier (B) increase the width of the barrier (C) increase the mass of the particle

do (A) only

Heisenberg's Uncertainty Principle of quantum mechanics

does none of the above

Heisenberg's Uncertainty Principle of quantum mechanics

does none of the above.

**Light from an argon laser strikes a diffraction grating that has 4,580 lines per cm. The central and first-order principal maxima are separated by 0.5 m on a wall 1.67 m from the grating. What is the wavelength of the laser light in nm? Enter an integer.

dsinθ = mλ. Here we cannot use the small angle approximation. (0.5m is notvery small compared to 1.67 m.) sinθ = 0.5/sqrt(0.52 + 1.672)=0.16 d = 1 cm/ 4580 = 10-2 m/4580=0.0000021834 m = 1 dsinθ = mλ 3.49*10^-7 m=Lambda Lambda=349 nm

Which of the statements below is true? a) Photons from a certain source have energy of 0.62 eV. This light is visible. b) Consider the photoelectric effect. The shorter the wavelength of the incoming light, the smaller is the cutoff frequency. c) Photons can never be destroyed. d) Consider the photoelectric effect. The greater the intensity of the incoming light, the smaller is the cutoff frequency. e) Consider the photoelectric effect. The greater the work function of the target, the greater is the cutoff frequency.

e) Consider the photoelectric effect. The greater the work function of the target, the greater is the cutoff frequency.

The configuration of the potassium atom is 1s2 2s2 2p6 3s2 3p6 4s1. Which of the following statements about potassium is true? a) Its 4s subshell is completely filled. b) Its n = 3 shell is completely filled. c) Its least tightly bound electron has l = 4. d) Its atomic number is 17. e) Its electron charge distribution or probability density is spherically symmetric.

e) Its electron charge distribution or probability density is spherically symmetric.

Which one of the following statements concerning rays is false? a) Rays point in the direction of the wave velocity. b) Rays are radial lines that originate from a point source of waves. c) Rays point outward from the wave source. d) Rays for a plane wave are parallel to each other. e) Rays are parallel to the wave front.

e) Rays are parallel to the wave front.

Which of the following is NOT correct for a simple magnifying glass? a) The image is virtual. b) The image is larger than the object. c) The object is closer to the lens than the focal point. d) The image is erect. e) The lens is diverging

e) The lens is diverging

Why does a soap bubble reflect virtually no light just before it bursts? a) This is an example of polarization by reflection. The light becomes p-polarized, but only s-polarized light is reflected. b) We have total internal reflection. The light keeps on reflecting inside the bubble until the bubble bursts. c) Light does not interact with a soap film. d) It absorbs all the light energy, so that it has enough energy to burst. e) The reflected light from the front and back surface destructively interfere, because the path difference is nearly zero, but the phase shift upon reflection differs by 180o for the two surfaces.

e) The reflected light from the front and back surface destructively interfere, because the path difference is nearly zero, but the phase shift upon reflection differs by 180o for the two surfaces.

The total energy of a particle is the sum of its kinetic energy and its potential energy. The zero of the potential energy can be chosen arbitrarily, so the potential energy can be negative. Classically forbidden regions are where a) a particle's total energy is equal its potential energy b) a particle's total energy is greater than its potential energy c) a particle's total energy is less than its kinetic energy. d) a particle's total energy is greater than its kinetic energy e) a particle's total energy is less than its potential energy

e) a particle's total energy is less than its potential energy

One of the predictions of Maxwell's theory of electromagnetic waves is that changing electric fields a) do not affect charged particles. b) produce streams of electrons. c) cause sparks. d) produce longitudinal waves. e) are the source of magnetic fields

e) are the source of magnetic fields

If you shoot a single photon through two slits to hit a screen it a) will hit anywhere that it can travel in a straight line from the gun to the screen. b) hits at a random location, with an equal probability of hitting anywhere on the screen. c) must hit at the maximum of the interference pattern. d) cannot hit in the middle because the block is in the way. e) has a chance of hitting anywhere on the screen, but on average a better chance at hitting where the interference pattern is brightest.

e) has a chance of hitting anywhere on the screen, but on average a better chance at hitting where the interference pattern is brightest.

Complete the following statement: An individual copper atom emits electromagnetic radiation with wavelengths that are a) the same as those of all elements. b) the same as other elements in the same column of the periodic table. c) unique to that particular copper atom. d) evenly spaced across the spectrum. e) unique to all copper atoms.

e) unique to all copper atoms.

Which of the following statements are false for a single diverging lens? (a) A diverging lens is thicker at the center than it is at the edges. (b) A diverging lens only produces virtual images. (c) A diverging lens can produce images which are both magnified and reduced in size. (d) A diverging lens only produces upright images. (e) A diverging lens has a negative focal length. (f) The images formed by a diverging lens can be located on either side of the lens relative to the object. Answers: a. (a), (b), (f) b. (a) c. (b) , (c), (f) d. (d), (e) e. (a), (c), (f)

e. (a), (c), (f)

The figure below shows different ways to connect a light bulb to a battery with wires. Which of the bulbs will light up? IMAGE PRACTICE 1 Answers: a. 1 b. 2 c. 3 d. 4 e. 5

e. 5

Two parallel light rays, initially in phase and having a 500 nm wavelength, reach a detector after one of the rays travels through a 10 cm long block of glass with an index of refraction of 1.5, while the other ray stays in air. The optical path difference between the two rays at the detector is a. 10 cm b. 750 nm c. 15 cm d. 1 cm e. 5 cm

e. 5 cm

Heisenberg's Uncertainty Principle of quantum mechanics a. is a mathematical formula that lets us calculate the momentum of an electron or other particle if we know its position. b. relates the energy and frequency of an electron or other particle. c. is a mathematical formula that lets us calculate the velocity of an electron or other particle if we know its energy. d. expresses a relationship between the wavelength and frequency of a photon or other particle. e. does none of the above.

e. does none of the above.

**A stream of electrons is of energy E is incident on a potential barrier of height U and thickness d. Even though U >> E, 5% of the electrons tunnel through the barrier. If the thickness of the barrier decrease to 0.28 d, what percentage of the electrons will tunnel through? Enter an integer.

e^-d/e^-0.28d =0.05/n 0.05/n = e^0.28d/e^0.d calculate n... multiply it with 100.

Two light nuclei come together and fuse forming a more massive nucleus. The mass of the product nucleus is less than the mass of the original nuclei. In this reaction Answers: nucleons are lost. the mass energy of final nucleus is equai to the mass energy of the initial nuclei. neutrons must be converted into protons. energy is released. energy is added.

energy is released.

**What is the wavelength (in nm) of a photon with energy E = 2.29 eV

energy of photon , E = 2.29 eV E = 2.29 * 1.6 * 10^-19 C E = 3.664 * 10-19 C let the wavelength be lamda E = h * c/lamda h = 6.626 * 10^-34 m^2 kg/s 3.664 * 10-19 = 6.626 * 10^-34 * 3 * 10^8 /( lamda) lamda = 5.43 * 10^-7 m lamda = 542.52 nm the wavelength of the photon is 542.52 nm

Which one of the following pairs of characteristics of light is best explained by assuming that light can be described in terms of photons? Answers: diffraction and the formation of atomic spectra polarization and the formation of line spectra photoelectric effect and the effect observed in Young's double slit experiment polarization and the photoelectric effect existence of line spectra and the photoelectric effect

existence of line spectra and the photoelectric effect

The Pauli exclusion principle Answers: forbids electrons and protons to interact. states that electrons have spin. prevents atoms from radiating and electrons from spiraling into the nucleus. explains the structure of the periodic table. excludes momentum and position from the uncertainty principle.

explains the structure of the periodic table.

A current in a long, straight wire produces a magnetic field. The magnetic field lines Answers: form circles that go around the wire. form circles that pass through the wire. are parallel to the wire. come in from infinity to the wire. go out from the wire to infinity.

form circles that go around the wire.

** with energy of 25 eV have a wavelength of ~0.25 nm. If we send these electrons through the same two slits (d = 0.44 mm) we use to produce a visible light interference pattern what is the spacing (in micrometer) between maxima on a screen 3.4 m away?

fringe width \beta = \lambda D/d \beta = 0.25x10^-9 x 3.4 / 0.44x10^-3 \beta = 1.93 x10^-6m spacing between maxima \beta = 1.93 micro meter

If you shoot a single photon through two slits to hit a screen it

has a chance of hitting anywhere on the screen, but on average a better chance at hitting where the interference pattern is brightest.

If you shoot a single photon through two slits to hit a screen it Answers: has a chance of hitting anywhere on the screen, but on average a better chance at hitting where the interference pattern is brightest. hits at a random location, with an equal probability of hitting anywhere on the screen. will hit anywhere that it can travel in a straight line from the gun to the screen. must hit at the maximum of the interference pattern. cannot hit in the middle because the block is in the way.

has a chance of hitting anywhere on the screen, but on average a better chance at hitting where the interference pattern is brightest.

A radioactive atom X emits a β− particle. The resulting atom

has an atomic number that is one more than that of X.

The initial fragments formed by fission

have more neutrons than protons.

The initial fragments formed by fission Answers: a. have more protons than neutrons. b. are a daughter nucleus and a alpha particle. c. have about the same number of neutrons and protons. d. are only neutrons. e. have more neutrons than proton

have more neutrons than protons.

**In 7,913 seconds, the number of radioactive nuclei decreases to 1/16 of the number present initially. What is the half-life (in s) of the material? Enter an integer.

here, let the half life be h the final number of radioactive present is 1/16 Since (1/2)^4 = 1/16, it takes 4 half lives for a material to decay to 1/16 of its initial value. So 4 * h = 8078 s h = 2019.5 s the half life of the material is 2020 s

What is the deBroglie wavelength (in nm) of a neutron (m = 1.67*10-27 kg) moving with a speed of 37 m/s? Enter a number with 2 decimal places.

https://d2vlcm61l7u1fs.cloudfront.net/media%2Fd04%2Fd04053dd-25d3-4d7b-9a39-455736a7a9d1%2Fphpx2QFt1.png

**The angular separation of the two components of a double star is 1.3 microradians, and the light from the double star has a wavelength of 567 nm. What is the diameter (in cm) of the smallest telescope mirror that will resolve the double star? Express your answer as an integer.

https://d2vlcm61l7u1fs.cloudfront.net/media%2Ffc1%2Ffc19d79c-13df-470b-a6ff-521cf229075f%2FphplJuUp5.png

An electron with an energy of 1000 eV has a de Broglie wavelength of 0.04 nm. If the energy of the electron decreased by a factor of 2, the wavelength will

increase by a factor of √2.

**A radioactive substance has a mean-life τ of 16 years. Assume we start with 118 kg of the substance. How many kg of the substance will be left after 19 years?

initial mass , m0 = 118 Kg helf life ,t0.5 = 16 years after 19 years for the mass left m = m0 * 0.5^(t/t0.5) m = 118 * 0.5^(19/13) m = 42.85 Kg the mass of substance left after 19 years is 42.85 Kg

A positive charge is moving with velocity v in a region with magnetic field B. What is the direction of the force on the particle? (IMAGE PRACTICE 1) Answers: There is no force on the particle. opposite the direction of v in the direction of v out of the page into the page

into the page

A magnet moving past an object will produce eddy currents in the object only if the object a) is made of magnetic material. b) is a paramagnetic material. c) is a liquid. d) is an insulator. e) is a conductor

is a conductor

Two stars that are close together are photographed through a telescope. Which situation would result in the most clearly separated images of the stars? Answers: one red and one blue star, the size of the lens does not matter large lens, red stars small lens, red stars small lens, blue stars large lens, blue stars

large lens, blue stars

The resolving power or resolution of a telescope is improved by using

larger diameter telescopes objectives & shorter wavelengths.

The resolving power or resolution of a telescope is improved by using Answers: a. lower frequency light. b. larger diameter telescopes objectives & longer wavelengths. c. mirrors instead of lenses. d. larger diameter telescopes objectives & shorter wavelengths. e. infrared light.

larger diameter telescopes objectives & shorter wavelengths.

The resolving power or resolution of a telescope is improved by using a) infrared light. b) larger diameter telescopes objectives & shorter wavelengths. c) larger diameter telescopes objectives & longer wavelengths. d) mirrors instead of lenses. e) lower frequency light

larger diameter telescopes objectives & shorter wavelengths.

Compared to the effective resistance of two resistors connected in series, the effective resistance of the same two resistors connected in parallel is Answers: either higher, lower, or the same, depending on the resistances of the individual resistors. lower. the same. higher.

lower.

Compared to the effective resistance of two resistors connected in series, the effective resistance of the same two resistors connected in parallel is Answers: lower. the same. either higher, lower, or the same, depending on the resistances of the individual resistors. higher.

lower.

**A thin film of oil (n = 1.42) is located on a smooth, wet pavement. When viewed perpendicular to the pavement, it appears to be predominantly red (637 nm). What is the minimum thickness of the oil film in nm?

minimum thickness for constructive interefrence , 2*t = lamda*n/2 t = lamda*n/4 = 637*10^-9*1.42/4 = 1.3135*10^-8 m = 13.135 nm

When a positive charge is released from rest in a uniform electric field, it will

move with constant acceleration

The electronic energy level of a certain system are given by En = E1*n2, where n = 1, 2, 3. Assume that transitions can occur between all levels. If one wanted to construct a laser from this system by pumping the n = 1 to n = 3 transition, which energy level or levels would have to be metastable?

n = 2 only

The electronic energy level of a certain system are given by En = E1*n2, where n = 1, 2, 3. Assume that transitions can occur between all levels. If one wanted to construct a laser from this system by pumping the n = 1 to n = 3 transition, which energy level or levels would have to be metastable? Answers: none n = 1 only n = 2 only n = 1 and n = 3 only n = 1, n = 2, and n = 3

n = 2 only

The electronic energy level of a certain system are given by En = E1*n2, where n = 1, 2, 3. Assume that transitions can occur between all levels. If one wanted to construct a laser from this system by pumping the n = 1 to n = 3 transition, which energy level or levels would have to be metastable? a) n = 1, n = 2, and n = 3 b) none c) n = 2 only d) n = 1 only e) n = 1 and n = 3 only

n = 2 only

Which of the transitions below can produce a spectral line in the visible region of the emission spectrum of hydrogen?

n = 4 --> n = 2

Of the following transitions in the hydrogen atom, the __________ transition results in the emission of the highest-energy photon. Answers: n = 1 → n = 6 n = 1 → n = 4 n = 3 → n = 6 n = 6 → n = 3 n = 6 → n = 1

n = 6 → n = 1

**An electron in a hydrogen atom makes a transition from the n = 2 to the n = 69 energy state. Determine the wavelength of the emitted photon (in nm).

n1 = 2 n2 = 69 let the wavelength emitted is L Using Bohr's formula for energy state transition h * c/wavelength = (13.6 * e) * (1/n1^2 - 1/n2^2) 6.626 *10^-34 * 3 *10^8/(L) = 13.6 * 1.602 *10^-19 * (1/2^2 - 1/69^2) solving for L L = 3.653 *10^-7 m = 365.3 nm the wavelength of light emitted is 365.3 nm

**Assume that in an atom the subshell n = 5, l = 3 is completely filled. How many electrons does this subshell hold?

n=5 l=3, then the orbital is f orbital mi= -3,-2,-1,0,1,2,3 There are 6 sub shells, each shell can hold 2 electrons. Therefore, a completely filled f orbital can occupy (7 x 2=14) 14 electrons.

Suppose you wanted to get rid of 12C nuclei and in the process release energy. In principle, this could be done by Answers: burning the carbon in an oxygen atmosphere. Even in principle this could not be done. nuclear fission into He nuclei. separating the nuclei into protons and neutrons. nuclear fusion.

nuclear fusion.

The middle of the first order maximum, adjacent to the central bright fringe in a double-slit experiment, corresponds to a point where the optical path length difference from the two slits is equal to

one wavelength.

The middle of the first order maximum, adjacent to the central bright fringe in a double-slit experiment, corresponds to a point where the optical path length difference from the two slits is equal to Answers: undefined. 1/2 wavelength. one wavelength. 1/4 wavelength. 0.

one wavelength.

The middle of the first order maximum, adjacent to the central bright fringe in a double-slit experiment, corresponds to a point where the optical path length difference from the two slits is equal to a) 1/2 wavelength. b) 0. c) one wavelength. d) undefined. e) 1/4 wavelength

one wavelength.

A beam of light A has twice the wavelength but the same intensity as a beam B. The number of photons that hit a given area in a given time, when illuminated by beam B is

one-half the number that hit when the area is illuminated by beam A.

A beam of light A has twice the wavelength but the same intensity as a beam B. The number of photons that hit a given area in a given time, when illuminated by beam B is a. one-quarter the number that hit when the area is illuminated by beam A. b. one-half the number that hit when the area is illuminated by beam A. c. twice the number that hit when the area is illuminated by beam A. d. the same as the number that hit when the area is illuminated by beam A. e. four times the number that hit when the area is illuminated by beam A.

one-half the number that hit when the area is illuminated by beam A.

A material is known to be an isotope of lead, although the particular isotope is not known. From such limited information, which of the following quantities can you specify: its atomic number, its neutron number, its atomic mass number?

only its atomic number

A material is known to be an isotope of lead, although the particular isotope is not known. From such limited information, which of the following quantities can you specify: its atomic number, its neutron number, its atomic mass number? Answers: a. only its atomic number b. only its neutron number c. only its atomic mass number d. its atomic number and its atomic mass number e. its neutron number and its atomic mass number

only its atomic number

A solid, conducting sphere of radius R is positively charged. Of the following distances from the center of the sphere, which location will have the greatest electric potential? (Take V = 0 at r = infinity.) a) r = 1.25 R. b) None of the above because the potential is constant. c) r = 2 R. d) r = 0 (center of the sphere) e) r = 1.1 R.

r=0

The nuclear force is

short range and charge independent.

The nuclear force is Answers: a. short range and charge independent. b. electromagnetic in nature. c. long range and charge independent d. long range and charge dependent e. short range and charge dependent.

short range and charge independent.

Bohr's model of the atom accounted for the spectra of

single-electron atoms.

Bohr's model of the atom accounted for the spectra of Answers: a. all atoms b. normal Helium atoms. c. many low atomic number atoms d. single-electron atoms. e. all noble gases.

single-electron atoms.

**A beam of green light is diffracted by a slit of width 0.55 mm. The diffraction pattern forms on a wall 2.06 m beyond the slit. The distance between the positions of zero intensity on both sides of the central bright fringe is 4.1 mm. What is the wavelength of the laser light in nm? Enter an integer.

slid width , d = 0.55 mm screen distance , D = 2.06 m Here , as distance between zero intensity points y = 2 * wavelength * D/d 4.1 *10^-3 = 2 * wavelength * 2.06/(0.55 *10^-3) solving for wavlength wavelength = 5.473 *10^-7 m = 547 nm the wavelength of the light wave is 547 nm

Electron diffraction proves Answers: that electrons are not point particles, but very smeared out entities. that we cannot know the electron's position as its moves through a slit and the direction of its momentum with absolute certainty. that electrons cannot be counted. that there are two kinds of electron, those that behave like particles, and those that behave like waves. that the electron is not a particle, but a wave.

that we cannot know the electron's position as its moves through a slit and the direction of its momentum with absolute certainty.

If the wavelength of the light were changed, and yellow light were passed through the slit,

the central bright region would become wider.

The figure shows the interference pattern obtained in a double-slit experiment using light of wavelength 500 nm. distance from both slits?

the middle slit

When a car is 500 m ahead of you, you see its tail lights as one long, red light. When the car is 100 m ahead of you, you see that the tail lights are actually several red lights placed close to each other. This is because

the pupil of your eye has a finite size.

White light passes through a diffraction grating and forms rainbow patterns on a screen behind the grating. For each rainbow pattern Answers: the red side is on the left, the violet side on the right. the red side is farthest from the center of the screen, the violet side is closest to the center. the red side is closest to the center of the screen, the violet side is farthest from the center. the red side is on the right, the violet side on the left.

the red side is farthest from the center of the screen, the violet side is closest to the center.

In your body Answers: there are about the same number of protons and neutrons. there are more electrons than protons. there are more protons than neutrons. there are more protons than electrons. there are more neutrons than protons.

there are more protons than neutrons.

Complete the following statement: An individual copper atom emits electromagnetic radiation with wavelengths that are

unique to all copper atoms.

Complete the following statement: An individual copper atom emits electromagnetic radiation with wavelengths that are Answers: a. evenly spaced across the spectrum. b. unique to that particular copper atom. c. unique to all copper atoms. d. the same as those of all elements. e. the same as other elements in the same column of the periodic table.

unique to all copper atoms.

The 'magnetic' quantum number 'm' of an electron in an atom is primarily

used to determine the orientation of the orbital.

The principal quantum number 'n' of an electron in an atom is primarily

used to identify the main energy level or shell.

An erect object is located between a concave mirror and its focal point. Its image is Answers: real, inverted, and larger than the object. virtual, erect, and larger than the object. virtual, erect, and smaller than the object. real, erect, and larger than the object. virtual, inverted, and larger than the object.

virtual, erect, and larger than the object.

The interference pattern from two closely spaced slits can only is only produced if single slit diffraction spreads out the light. The graph below shows a double slit interference pattern. The ideal two-slit intensity pattern is multiplied by intensity of the single slit diffraction pattern. If this pattern was obtained with two slits spaced by d = 0.1 mm, what was the slit width w?

w = d/4

The interference pattern from two closely spaced slits can only is only produced if single slit diffraction spreads out the light. The graph below shows a double slit interference pattern. The ideal two-slit intensity pattern is multiplied by intensity of the single slit diffraction pattern. If this pattern was obtained with two slits spaced by d = 0.1 mm, what was the slit width w? Image7.gif

w = d/4

**A laser emits photons continuously at a rate of 2.7*10^16/s. If the wavelength of the photons is 539 nm, what is the laser power in mW? Enter a number with one digit behind the decimal point.

wavelength , lamda = 539 nm = 5.39 * 10^-7 m the energy of each photon , E = h * c/lamda the laser power , P = n * E P = 2.7 * 10^16 * 6.634 * 10^-34 * 3 * 10^8 /( 5.39 * 10^-7 ) W P = 9.97 * 10^-3 W = 9.97 mW the power of laser is 9.97 mW

The white circles in the figure represent minima of the diffraction pattern formed when a bright point-source, for example a distant star, is viewed through a small circular opening. In accordance with Rayleigh's criterion, the closest the central maximum of another point source could lie in this pattern and still be barely resolvable IMAGE PRACTICE TEST 1 Answers: would be at point A. must lie outside all discernible diffraction rings. would be at point D. would be at point B. would be at point C.

would be at point B.

The wave function ψ(x,t) for a neutron is shown at some time t. Around which point is the neutron most likely to be found at time t?

x = xC

The wave function ψ(x,t) for a neutron is shown at some time t. Around which point is the neutron most likely to be found at time t? Image5.gif

x = xC

The wave function ψ(x,t) for a neutron is shown at some time t. Around which point is the neutron most likely to be found at time t? x = xC There is no most likely place. x = xA x = xB x = 0

x = xC

An energy eigenfunction ψ(x) for an electron in the potential energy well pictured below is shown. Around which point is the electron most likely to be found?

x= D

Which statement is false? Quantum mechanics predicts that

you can never know the energy and the magnitude of the angular momentum exactly at the same time.

Which statement is false? Quantum mechanics predicts that Answers: a. all angular momentum is quantized in magnitude as well as in direction. b. confined particles can only have certain discrete energies. c. it is impossible to know the exact energy of a particle and to be able to track the particle. d. if, in principle, you can only observe a particle for a short time, you cannot know its energy exactly. e. you can never know the energy and the magnitude of the angular momentum exactly at the same time.

you can never know the energy and the magnitude of the angular momentum exactly at the same time.

Identify in order the decay processes in the decay series shown below

α, β-, β-, α

Identify in order the decay processes in the decay series shown below. (IMAGE PRACTICE TEST 1) Answers: α, γ, γ, α β-, α, β+, β- α, β-, β-, α α, β+, β+, α α, β-, β+, γ

α, β-, β-, α

It is necessary to coat a glass lens with a nonreflecting layer. If the wavelength of the light in the coating is λ, the best choice is a layer of material having an index of refraction between those of glass and air and a thickness of

λ/4

Light of wavelength λ1 illuminates a double slit, and interference fringes are observed on a screen behind the slits. When the wavelength is changed to λ2, the fringes move closer together. What is the relationship between λ1 and λ2.

λ1 is larger than λ2.


Kaugnay na mga set ng pag-aaral

Chapter 5 & 6 Health Infectious and Noninfectious Disease

View Set

(Chemistry H) Benchmarks & Finals: Quarter 3 Benchmark

View Set

Past simple and past participle irregular verbs

View Set

Іспит 📚 Бази Даних

View Set

Factors that will affect insurance cost and basic coverage terms

View Set

NFA 201: Unit 2 SmartBook- Nutrition Information Fact or Fiction?

View Set

Foundations of Western Culture14-27

View Set

Anatomy and Physiology: Chapter 6,7,&8 Review

View Set